Question type approach
How would you approach this question besides plugging in every answer choice, without having a gu...
Jasmin1 06:58PM
  • June 2019 LSAT
  • SEC4
  • Q15
1
Reply
difference between A and B?
What is the difference between answers A and B? They seem to be stating the same thing.
shaase 06:53PM
  • June 2019 LSAT
  • SEC4
  • Q17
1
Reply
Why not A?
A fits the rules i think but why is it incorrect?
etchun 06:52PM
  • June 2019 LSAT
  • SEC4
  • Q18
1
Reply
Time management
How do you recommend efficiently doing this question? Going through each scenario seems like a ti...
Matt9 06:50PM
  • June 2019 LSAT
  • SEC4
  • Q21
1
Reply
New question
Please explain the difference between a flawed argument vs a valid argument
Bradleybraaten Saturday at 08:15PM
  • June 2019 LSAT
  • SEC4
  • Q2
1
Reply
Section 4 Q 18
Why cant A be the correct answer. Since we have 3 slots available for P, Q; either P or Q can app...
Bola on August 24 at 12:13AM
  • June 2019 LSAT
  • SEC4
  • Q18
3
Replies
Why can't H be first?
Matt gets a deduction that H can't be first due to the 3rd rule, however if H is first I is first...
Jalvarez on April 15, 2023
  • June 2019 LSAT
  • SEC4
  • Q11
2
Replies
Golar
I thought it was H since there are more restrictions on that one.
denleybishop on May 28, 2022
  • June 2019 LSAT
  • SEC4
  • Q15
1
Reply
Rule 4
Hi, I wrote rule 4 as... H(before 3) --> I (1). The parentheses represent a subscript by the way...
Abigail-Okereke on March 3, 2022
  • June 2019 LSAT
  • SEC4
  • Q11
1
Reply
Help
It took me 11 mins to complete this game. Are there any tips on how I can move faster. My exam da...
Abigail-Okereke on March 3, 2022
  • June 2019 LSAT
  • SEC4
  • Q6
1
Reply
Helpp
I understand why B is right buy someone could please explain why D is wrong and how B is better t...
Maria-Marin on February 8, 2022
  • June 2019 LSAT
  • SEC4
  • Q20
7
Replies
Layout
What is the layout please of game and inferences that should be made
king on January 20, 2022
  • June 2019 LSAT
  • SEC4
  • Q23
1
Reply
Why only 2
Why don’t the rules say that fs or sf come before P?
DalilaPando on January 14, 2022
  • June 2019 LSAT
  • SEC4
  • Q6
1
Reply
Set-Up
Could you show the game set up? I didn't see the explanation for it.
mmanetti on August 9, 2021
  • June 2019 LSAT
  • SEC4
  • Q1
1
Reply
Correct Answer
Why is the correct answer not 4? I know that T can't be second because of the 3rd rule. What othe...
mmanetti on July 22, 2021
  • June 2019 LSAT
  • SEC4
  • Q9
2
Replies
Explanation
Can someone please explain the right answer?
zia305 on July 17, 2021
  • June 2019 LSAT
  • SEC4
  • Q16
4
Replies
Please answer
Why not e?
angelasargent on July 9, 2021
  • June 2019 LSAT
  • SEC4
  • Q14
5
Replies
Ans CHOICE A and D
Hi, So I chose G because of the G and R pair, and it would still determine the same, so why would...
ankita96 on December 17, 2020
  • June 2019 LSAT
  • SEC4
  • Q15
4
Replies
Question 9 - Time trap?
How should we efficiently get to this answer?
Matt9 on November 24, 2020
  • June 2019 LSAT
  • SEC4
  • Q9
2
Replies
Consecutively on the LSAT
What does consecutively on the LSAT mean? Does this that F & G are next to each other and it coul...
Kirsten on November 24, 2020
  • June 2019 LSAT
  • SEC4
  • Q9
1
Reply